12

Nếu là thường xuyên, nó có theo là thường xuyên không?A 2A2 AA

Nỗ lực của tôi về một bằng chứng:

Có, vì mâu thuẫn cho rằng AA không thường xuyên. Rồi Một 2 = Một MộtA2=AA .

Vì việc ghép hai ngôn ngữ không thường xuyên không thường xuyên nên A 2A2 không thể đều đặn. Điều này mâu thuẫn với giả định của chúng tôi. Vậy AA là thường xuyên. Vậy nếu A 2A2 là thường xuyên thì AA là thường xuyên.

Bằng chứng có đúng không?

Chúng ta có thể khái quát điều này thành A 3A3 , A 4A4 , v.v ... không? Và cũng nếu A *A là thường xuyên sau đó AA không cần phải thường xuyên?

Ví dụ: Một = { 1 2 i | i 0 }A={12ii0} là không thường xuyên nhưng A *A là thông thường.


2
Bằng chứng đầu tiên thực hiện một bước nhảy lớn. Bằng chứng của bạn về việc A không thường xuyên ngụ ý A 2 không thường xuyên là gì? Chứng minh rằng đúng có thể dẫn bạn đến trực giác để giúp trả lời phần còn lại của câu hỏi, nếu thực sự đó là sự thật. AA2
Dave Clarke

@DaveClarke Chỉnh sửa bằng chứng.
akshay

3
Làm thế nào bạn quản lý để đánh vần "Tôi đúng?" cách đó rất hấp dẫn. Như một lời khuyên chung: khi hàng trăm người đọc những gì bạn đã viết, sự quyết đoán chung đòi hỏi bạn phải chú ý đến cách bạn viết ... ;-)
Andrej Bauer

6
@AndrejBauer OP có thể là một người không phải là người nói tiếng Anh bản địa và có thể chưa có cơ hội được hướng dẫn về tiếng Anh chính thức. Đây không phải là lý do để làm nản lòng bất cứ ai, mặc dù nó có thể hữu ích để sửa chúng.
Yuval Filmus

Câu trả lời:


28

Hãy xem xét định lý bốn hình vuông của Lagrange . Nó nói rằng nếu B = { 1 n 2 | n 0 } thì B 4 = { 1 n | n 0 } . Nếu B 2 đều đặn, lấy A = B khác lấy A = B 2 . Dù bằng cách nào, điều này chứng tỏ sự tồn tại của A không đều mà A 2 là thường xuyên.B={1n2|n0}B4={1n|n0}B2A=BA=B2AA2


Tôi không hiểu bằng chứng này; bạn có thể xây dựng một chút?
G. Bạch

2
Lý giải điều này (đẹp) chứng minh: Chúng tôi có mà B R E G , và B 4R E G . Quan sát rằng B 4 = ( B 2 ) 2 . Bây giờ, nếu B 2R E G , sau đó bằng cách tham gia một = B chúng ta có một phản ví dụ, và nếu B 2R E G sau đó bằng cách tham gia một = B 2 chúng tôi có một phản ví dụ. BREGB4REGB4=(B2)2B2REGA=BB2REGA=B2
Shaull

1
Tuyệt đẹp.
vonbrand

3
@YuvalFilmus, thực sự, nhưng tôi không có bằng chứng và tôi không muốn để lại bất kỳ nghi ngờ nào. Bây giờ tôi dường như đã tìm thấy một. "Một số n là tổng của hai hình vuông khi và chỉ khi tất cả các thừa số nguyên tố có dạng 4 k + 3 thậm chí còn có số mũ trong hệ số nguyên tố của n ." Gọi n là chiều dài bơm. Xét w = ( n ! ) 2 . Đặt p là số nguyên tố có dạng 4 k + 3 và gọi m là độ dài chúng ta chọn để bơm. Sau đó, w + ( p - 1 )n4k+3nnw=(n!)2p4k+3mwmm=pwcó số mũ lẻ trênpvà do đó không nằm trongB2. w+(p1)wmm=pwpB2
Karolis Juodelė

1
@ JonasKölker, đồng ý.
Karolis Juodelė

8

Dưới đây là một ví dụ về một ngôn ngữ không phải tính toán Một ví dụ mà Một 2 = Σ * . Đi bất kỳ không tính toán K (biểu diễn dưới dạng một tập hợp các số, ví dụ như các mã máy Turing mà dừng lại), và xác định A = { w Σ * : | w | 4 k  cho tất cả  k K } . Vì vậy, một chứa tất cả các từ khác hơn so với những chiều dài 4 k đối với một số k K . Nếu AAA2=ΣK

A={wΣ:|w|4k for all kK}.
A4kkKAđã được tính toán sau đó bạn có thể tính K : đã cho k , xác định xem 0 4 k (có nghĩa là 4 k số 0) có ở A hay không. Vì chúng tôi giả sử K không thể tính toán được, A cũng phải không tính toán được.Kk04k4kAKA

Khẳng định: Một 2 = Σ * . Đặt w là bất kỳ từ có độ dài n . Nếu n không phải là một sức mạnh của 4 , sau đó w A và chữ rỗng là trong một , vì vậy w A 2 . Nếu n là lũy thừa 4 thì n / 2 không phải là lũy thừa 4 . Viết w = x y , trong đó | x | = | y | = n /A2=Σwnn4wAAwA2n4n/24w=xy2 . Cả hai x , y A để w = x y A 2 .|x|=|y|=n/2x,yAw=xyA2


1
Đối với người mới bắt đầu, một bản phác thảo bằng chứng cho " Một điều không thể giải quyết" có thể theo thứ tự. Ngoài ra, một trở ngại nhỏ có thể là bạn sử dụng K làm ngôn ngữ chính thức như một bộ số (công bằng, giả sử ngữ nghĩa phù hợp cho K , nhưng có thể không quen thuộc). Nếu không, ý tưởng rất tốt đẹp. AKK
Raphael

2

Bằng chứng của bạn vẫn tạo ra một bước nhảy lớn (lập luận rằng việc ghép các ngôn ngữ không thông thường là không thường xuyên).

Nếu phỏng đoán Goldbach là đúng, thì câu trả lời cho câu hỏi là không: Hãy xem ngôn ngữ không chính quy A = { 1 p : p  là số nguyên tố } . Sau đó, theo phỏng đoán Goldbach, A 2 = { 1 2 k : k > 1 } , là thường xuyên.A={1p:p is a prime}A2={12k:k>1}

Điều này không giải quyết hoàn toàn câu hỏi, nhưng nó đưa ra bằng chứng mạnh mẽ rằng câu trả lời là không (nếu không thì phỏng đoán Goldbach là sai). Tuy nhiên, câu trả lời có thể rất khó để chứng minh, nếu đây là ví dụ duy nhất được biết đến.


Chúng ta có thể kết luận gì về câu hỏi?
akshay

Giả sử phỏng đoán Goldbach -if A 2 là thường xuyên, hơn A vẫn có thể không thường xuyên. Vì vậy: chứng minh rằng câu trả lời là "có" sẽ có nghĩa là phỏng đoán Goldbach là sai (không chắc). A2A
Shaull

2
Với sự có mặt của bằng chứng "thực", tôi không nghĩ sử dụng một phỏng đoán chưa được chứng minh là công bằng. Có lẽ kết nối là thú vị cho một số?
Raphael

Thật vậy, sau những câu trả lời sau đây, điều này là dư thừa. Tuy nhiên, bạn có thể thấy một sự phát triển toán học tốt đẹp ở đây: một câu trả lời dựa trên một phỏng đoán nổi tiếng, sau đó là một câu trả lời liên quan (sử dụng định lý Lagrange), dựa trên một ý tưởng tương tự (phân tách một số thành một tổng).
Shaull

1
Trong thực tế nếu bạn sử dụng các số nguyên tố và bán đảo, bạn có thể sử dụng định lý Chen .
sdcvvc

2

Yêu cầu là sai.

Đặt D là ngôn ngữ không thông thường là "thưa thớt": nếu x D thì bất kỳ y D nào khác đều thỏa mãn | y | > 4 | x | (hoặc | x | > 4 | y | ) . Không quá khó để thấy rằng nhiều ngôn ngữ không thông thường có thể thưa thớt.DxDyD|y|>4|x||x|>4|y|

Bây giờ xác định A = Σ *D . Từ tính chất đóng (bổ sung), A phải không thường xuyên.A=ΣDA

Tuy nhiên, Một 2 = Σ *A2=Σ       (bạn có thể thấy tại sao?)

Tôi nghĩ | y | > 2 | x | là đủ, nhưng có thể gây ra một số trường hợp cạnh khó chịu. | y | > 2 | x | + 2 là đủ, mặc dù vậy, hãy dùng | y | > 4 | x | để được an toàn|y|>2|x||y|>2|x|+2|y|>4|x|


Tôi thích làm thế nào câu hỏi này ngày càng có nhiều bằng chứng tầm thường. Ý tưởng về độ thưa thớt của bạn có thể được đơn giản hóa hơn nữa theo yêu cầu 1 A1A1kA1k1A1kA1k1A.
Karolis Juodelė

2

Take any nonregular X1X1 and define A={1}{12x:xN}{12x+1:1xX}A={1}{12x:xN}{12x+1:1xX}.

It is easy to see AA is nonregular, while A2=1A2=1.


2

Let UNUN be any undecidable set, let I={2u+1uU}{0,2,4,}I={2u+1uU}{0,2,4,} and let L={aiiI}L={aiiI}. LL is undecidable so it certainly isn't regular. But L2={a2nnN}{annminI}L2={a2nnN}{annminI}, which is regular because its complement is finite.


0

Another example, from a question that was marked as a duplicate of this, is to consider the non-regular language {akm is composite}{akm is composite}. Any even number n8n8 is the sum of n4n4 and 44, which are both composite; any odd number n13n13 is the sum of n9n9 and 99, which are both composite (n9=2m for some m2). Therefore, A2={a8,a10}{akk12}, which is regular because it's co-finite (it's the complement of {ϵ,a,aa,,a6,a7,a9,a11}).

Khi sử dụng trang web của chúng tôi, bạn xác nhận rằng bạn đã đọc và hiểu Chính sách cookieChính sách bảo mật của chúng tôi.
Licensed under cc by-sa 3.0 with attribution required.